#17 Rosh Review

Réussis tes devoirs et examens dès maintenant avec Quizwiz!

Question: What common infection is known to trigger eruptive, inflammatory psoriasis?

Answer: Streptococcal Pharyngitis. Rapid Review Psoriasis Silvery scales Extensor surfaces Nail pitting Auspitz sign: scale removal produces blood droplets Arthritis

Question: Is the presence of one eosinophil in the CSF considered normal or abnormal?

Answer: The presence of even one eosinophil in the CSF is considered abnormal. Be suspicious in such cases of a parasitic infection. Rapid Review Viral Meningitis Enteroviruses CSF: normal glucose, mildly elevated protein (< 200 mg/dL), lymphocytic pleocytosis Rx: supportive

Question: At what penile length do you diagnose micropenis?

Answer: Less than 2 cm.

Question: Why should phenytoin be administered as an infusion over 45-60 minutes as opposed to a bolus?

Answer: The diluent in phenytoin causes hypotension if given rapidly. Rapid Review Febrile Seizure - Simple Patient will be a child 6 mos - 5 yrs Complaining of a single generalized seizure lasting < 15 mins Most commonly caused by rapid rise in temperature Treatment is supportive care

Question: What are the 3 most commonly identified bacterial agents in acute otitis media?

Answer: Streptococcus pneumoniae, Haemophilus influenzae, and Moraxella catarrhalis. Rapid Review Acute Otitis Media Patient will be an infant or young child Complaining of ear pain, fever, URI symptoms PE will show TM erythema and decreased mobility of TM Most commonly caused by viral > bacterial (S. pneumoniae most common) Treatment is amoxicillin

Question: What is the vector that spreads malaria?

Answer: The female Anopheles mosquito. Rapid Review Malaria P. falciparum (deadliest), P. ovale, P. vivax, P. malariae P. ovale, P. vivax: hepatic phase Anopheles mosquito Immigrant, traveler Irregular fevers, diaphoresis P. falciparum: cerebral malaria, Blackwater fever Uncomplicated, no resistance areas Rx: chloroquine Complicated, P. falciparum rx: quinidine + doxycycline

Question: What is the treatment for absence seizures?

Answer: The most common medications used are ethosuximide and valproic acid. Rapid Review Absence Seizures (petit mal) Patient will be 5 - 10 y/o Complaining of sudden mental status alteration without motor activity, blank stare EEG would show symmetric 3-Hz spike and wave activity Treatment is ethosuxamide Comments: no aura or postictal state

Question: What type of immune complex is associated with Henoch-Schönlein Purpura?

Answer: IgA. Rapid Review Henoch-Schonlein Purpura (HSP) Patient will be 4-12 years old With a history of recent URI Complaining of abdominal pain, arthralgia, and a rash that began on buttocks and lower extremities PE will show maculopapular rash (palpable purpura) that is non-pruritic Most commonly caused by IgA mediated vasculitis Treatment is supportive care Comments: Complications include nephropathy, intussusception

Question: Headaches associated with a brain tumor are classically worse during what time of day?

Answer: In the morning when intracranial pressure is highest after a night of recumbent positioning. Rapid Review Brain Tumors Most common: metastases Chronic headache worse upon awakening

Question: Patients with Fragile X syndrome are predisposed to which cardiac abnormality?

Answer: Mitral valve prolapse.

Question: What is the most common benign cause of a primary respiratory alkalosis?

Answer: Hyperventilation. Rapid Review Metabolic Acidosis pH <7.35 + HCO3- <24 ​Normal anion gap (AG): Hyperalimentation, Addison Disease, RTA, Diarrhea, Acetazolamide, Spironolactone, Saline infusion (HARDASS) ↑ AG: Methanol, Uremia, DKA, Propylene glycol, Iron or INH, Lactic acidosis, Ethylene glycol, Salicylates (MUDPILES) Tachypnea Winter's formula: PCO2 = 1.5(HCO3-) + 8 +/- 2 ​PCO2 < Winter's formula calculation = respiratory alkalosis PCO2 > Winter's formula calculation = respiratory acidosis

Question: How significant is the threat to male fertility resulting from testicular involvement as part of mumps virus disease?

Answer: Mumps orchitis rarely leads to sterility, but it may contribute to subfertility. It can also lead to defects in sperm numbers and movement.

Question: What physical health problems are associated with bulimia?

Answer: Electrolyte disturbances, loss of dental enamel, and esophageal tears. Rapid Review Bulimia Recurrent binge eating Overconcern with body shape/weight Weight: normal or above normal Submandibular gland enlargement Calluses over dorsal aspect of fingers Dental enamel erosion Rx: cognitive behavioral therapy (first-line), SSRIs (second-line)

Question: What is the most common pathogen implicated in bacterial vulvovaginitis?

Answer: Group A β-hemolytic streptococcus.

Question: What type of voice is associated with epiglottitis?

Answer: Hot potato voice. Rapid Review Epiglottitis Patient will be complaining of rapid onset of fever and dysphagia PE will show patient leaning forward, drooling, inspiratory stridor Imaging will show "thumbprint" sign Most commonly caused by H. influenzae, Streptococcus Treatment is IV antibiotics and airway management

Question: What thoracic pathologies should always be considered in patients with abdominal pain?

Answer: Myocardial infarction (or ischemia), pneumonia, pulmonary embolism, pericarditis and myocarditis. Rapid Review Abdominal Pain in the Elderly Poor historians ↓ PE reliability Polypharmacy 50% admitted 33% have surgery 10% mortality

Question: What is the success rate of enema reductions in patients with intussusception?

Answer: 80-95%. Rapid Review Intussusception (Telescoping Bowel) Patient will be a child 5 months - 3 years old Complaining of colicky abdominal pain, vomiting and bloody stools (currant jelly) Diagnosis is made by ultrasound ("target sign") Most commonly caused by a tumor or meckel's diverticulum Treatment is air/contrast enema

Question: What is the risk of anaphylactic reaction to cephalosporins in a penicillin allergic patient?

Answer: < 10%.

Question: What are Osler's nodes?

Answer: A sign of subacute bacterial endocarditis, these are the result of immune complex deposition, leading to tender nodules in the digit pads. Rapid Review Endocarditis Patient will be complaining of fever, rash, cough and myalgias PE will show Fever, Roth spots, Osler nodes, Murmur, Janeway lesions, Anemia, Nailbed hemorrhages, Emboli (FROM JANE) Diagnosis is made by echocardiography and Duke's criteria Most commonly caused by: IVDA: S. aureus, tricuspid Native valve: Streptococci, mitral Treatment is antibiotics Comments: GI malignancy: S. bovis

Question: What other diagnoses are more common amongst children with eczema?

Answer: Allergic rhinitis, asthma, and food allergies. Rapid Review Atopic Dermatitis (Eczema) Patient with a history of asthma or hay fever Complaining of itchy, scaly rash on arms, often worse in the winter PE will show thick, leathery, hyperpigmented areas on flexor surfaces Treatment is topical corticosteroids, lubricating ointments

Question: Approximately one-third of patients with Wolff-Parkinson-White develop what tachydysrhythmia?

Answer: Atrial fibrillation, and if atrial fibrillation conducts rapidly down the bypass pathway, it can degenerate into ventricular fibrillation.

Question: Which dysrhythmia is commonly associated with mitral regurgitation?

Answer: Atrial fibrillation. Rapid Review Mitral Regurgitation Patient with a history of ischemic heart disease, endocarditis, MI, trauma Complaining of dyspnea PE will show blowing holosystolic mumur, best heard at apex with radiation to axilla, pulmonary edema, cardiogenic edema Diagnosis is made by echo Treatment is nitroprusside, dobutamine, intraaortic balloon pump, emergency surgery

Question: For what indication is oral vancomycin most useful?

Answer: C. difficile colitis.

Question: What is the medical treatment of symptomatic premature atrial contractions?

Answer: Calcium-channel blockers or beta-blockers. Rapid Review Premature Atrial Contractions Extra beats that originate outside sinus node Originate from ectopic atrial pacemaker Different morphology from sinus P waves Appears interspersed throughout an underlying rhythm

Question: What is the Trousseau sign?

Answer: Carpal spasm when the blood pressure cuff is inflated to pressures above systolic pressure. Rapid Review Hypocalcemia Serum Ca2+ <8.5 mg/dL Blood transfusions Seizures, paresthesias Chvostek's sign (facial muscle contraction) Trousseau sign (carpopedal spasm) Hyperreflexia ↑ QT interval

Question: What are the two most common organisms responsible for epididymitis under the age of 35?

Answer: Chlamydia trachomatis and Neisseria gonorrhea. Rapid Review Testicular Torsion: Patient will be a young male Complaining of intense scrotal pain PE will show exquisite tenderness of the testicle and no cremasteric reflex Diagnosis is made by ultrasound with Doppler Treatment is manual detorsion or surgical

Question: Name two complications of chronic eustachian tube dysfunction?

Answer: Chronic otitis media and tympanic membrane perforation.

Question: Pathologic phimosis can be associated with which endocrinopathy?

Answer: Diabetes mellitus, especially if not treated or undertreated, as chronic glycosuria can cause chronic foreskin infection. Rapid Review Phimosis Inability to retract foreskin behind glans Rx: hygiene, topical steroids Rx if vascular compromise: dorsal slit Definitive rx: circumcision

Question: What are the risk factors for infantile strabismus?

Answer: Down syndrome, cerebral palsy, craniofacial dystocia, low birth weight and family history. Rapid Review Strabismus Esotropia: in Exotropia: out Cover-uncover exam Light reflex test

Question: Is antibiotic prophylaxis indicated in patients with isolated aortic stenosis?

Answer: No. Rapid Review Aortic Stenosis Patient will be older With a history of diabetes, hypertension Complaining of dyspnea, chest pain, syncope PE will show crescendo-decrescendo systolic murmur that radiates to the carotids, paradoxically split S2, S4 gallop Most commonly caused by degenerative calcification Treatment is aortic valve replacement Comments: murmur decreases with valsalva

Question: How reliable are ESR and CRP in ruling out a septic joint?

Answer: Not very. ESR is normal in up to 30% of patients and CRP is normal in up to 12% of patients. Rapid Review Septic Arthritis Age <35: N. gonorrhea S. aureus most common overall Hematogenous spread Fever, pain, ↓ ROM Knee (most common) Arthrocentesis (WBC >50,000 with >75% PMNs) IV ABX, surgical washout

Question: What urinalysis finding is present in patients with marked hyperlipidemia from nephrotic syndrome?

Answer: Oval fat bodies, lipid deposits in sloughed renal tubular epithelial cells, occur with marked hyperlipidemia. Rapid Review Nephrotic Syndrome Proteinuria > 3.5 g/24 hr Hyperlipidemia Hypercoagulability (renal vein thrombosis) Fatty casts Pitting edema Minimal change disease: children, preceded by URI, rx: steroids Focal segmental glomerulosclerosis: African-Americans, HIV/IVDA Membranous nephropathy: Caucasians, HBV, HCV, SLE, gold, penicillamine , malignancy Type 1 MPGN: HBV, HCV Type 2 MPGN: ↓ C3 Diabetic glomerulonephropathy

Question: When are patients with herpes zoster considered to no longer be infectious?

Answer: Patients are no longer considered infectious once the herpes zoster lesions are crusted over. Rapid Review Herpes Zoster Age, immunodeficiency Reactivation of latent VZV in dorsal root ganglion Grouped vesicles on an erythematous base, dermatomal Postherpetic neuralgia, Ramsay Hunt syndrome, zoster ophthalmicus Acyclovir

Question: In which patients should NSAIDs be avoided?

Answer: Patients with acid peptic ulcer disease, renal insufficiency, diabetes and a history of allergic reaction to NSAIDs. Rapid Review Lumbago Acute nonspecific low back pain Lack Red flags Continue daily activities as tolerated Physical therapy, NSAIDs, acetaminophen, opioids

Question: What other treatment would you consider for this patient with lumbago?

Answer: Physical therapy, acetaminophen or NSAIDs, and muscle relaxers. Rapid Review Lumbago Lack red flags Continue daily activities Physical therapy, analgesia

Question: What finding is seen on synovial fluid analysis in pseudogout?

Answer: Positively birefringent, rhomboid-shaped crystals. Rapid Review Gout Patient will be a middle-aged man Complaining of acute onset of pain in the first MTP (Podagra) Labs will show needle-shaped crystal with negative birefringence Most commonly caused by uric acid crystals Treatment is: Acute: NSAID's Chronic: allopurinol or colchicine Comments: can be triggered by loop and thiazide diuretics

Question: What drug can be administered to reopen a recently closed ductous arteriosus?

Answer: Prostaglandin E1. Rapid Review Tetralogy of Fallot Most common cyanotic congenital heart disease Pulmonic stenosis + RVH + VSD + overriding aorta Pulmonic stenosis determines degree of cyanosis Tet spells (crying/feeding → ↑ RV outflow obstruction) CXR: "boot-shaped" heart Acute cyanosis rx: place child in squatting or knee to chest position (↑ SVR → ↓ shunting), morphine, O2 Definitive rx: surgery

Question: What electrocardiogram finding is associated with macrolide antibiotic use?

Answer: QT interval prolongation Rapid Review Pyloric Stenosis Patient will be 2 - 6 weeks old Complaining of non-bilious projectile vomiting after feeding and early satiety PE will show RUQ olive-like mass (hypertrophied pylorus) Labs will show hypochloremic hypokalemic metabolic alkalosis Diagnosis is made by ultrasound or UGI series (string sign) Treatment is surgical

Question: Does LDH elevation have a greater sensitivity or specificity for Pneumocystis jiroveci pneumonia?

Answer: Serum LDH has a greater sensitivity (approximately 90%) in Pneumocystis jiroveci pneumonia but a poor specificity. Rapid Review Pneumocystis Pneumonia (PCP) Patient with a history of HIV Complaining of gradual onset of non-productive cough Labs will show CD4 < 200, increased LDH CXR will show bilateral infiltrates (bat wing pattern) Most commonly caused by Pneumocystis jirovecii Treatment is TMP-SMX

Question: What causes obstructive sleep apnea?

Answer: Soft tissue collapse in the pharynx. Rapid Review Obstructive Sleep Apnea Patient will be obese With a history of allergies Complaining of apneic episodes while sleeping, snoring and daytime sleepiness PE will show enlarged tonsils Diagnosis is made by sleep study Treatment is weight loss, CPAP Comments: can lead to pulmonary hypertension and cor pulmonale (right ventricular hypertrophy)

Question: What is the most common infectious agent in acute osteomyelitis?

Answer: Staphylococcus aureus. Rapid Review Osteomyelitis Adults: contiguous spread Children: hematogenous spread Most common: S. aureus Sickle cell: Salmonella Cat/dog bites: Pasteurella multocida Plain films: periosteal elevation or bony erosions Bone scan, MRI Long term ABX

Question: Which medication is recommended for patients with atrophic vaginitis and dyspareunia who do not want to use a vaginal route of medication delivery?

Answer: The selective estrogen receptor modulator (SERM) ospemifene. Rapid Review Atrophic Vaginitis Patient will be a postmenopausal woman Complaining of dyspareunia, dryness, bleeding, itching PE will show pale, dry, shiny epithelium Most commonly caused by decrease in estrogen Treatment is lubricants, moisturizers, topical estrogen (2nd line)

Question: What test is used to diagnose a hydrocele?

Answer: Transillumination. Rapid Review Epididymitis Patient will be complaining of gradual onset unilateral scrotal pain PE will show increased color flow on doppler, relief with testicular elevation (Prehn's sign) Most commonly caused by < 35 yo: C. trachomatis, N. gonorrhea > 35 yo: E. coli, Pseudomonas Treatment is < 35 y/o ceftriaxone/doxycycline, > 35 y/o ciprofloxacin

Question: The "wire brush" feel is characteristic of what hair loss condition?

Answer: Trichotillomania. Rapid Review Alopecia Areata Patient with a history of an autoimmune disorder Complaining of hair loss PE will show patches of smooth, non-scarring hair loss with patches of smaller hairs termed "exclamation hairs" Most common cause is autoimmune Treatment is intralesional corticosteroids

Question: Which is more serious, type I (Mobitz I/Wenckebach) or type II (Mobitz II) second-degree AV block?

Answer: Type II, which is more associated with complete heart block and cardiac arrest. Rapid Review Second Degree Heart Block, type I (Wenkebach/Mobitz I) Progressive prolongation of PR interval until one QRS complex is dropped

Question: What is the utility of a Tzanck test?

Answer: Tzanck tests are no longer recommended due to poor sensitivity. Rapid Review Male Genital Lesions Herpes: painful, shallow, cluster of vesicles Syphilis: painless chancre LGV: C. trachomatis, painless ulcer, inguinal lymphadenopathy Chancroid: H. ducreyi, painful chancres, inguinal bubo Granuloma inguinalea: K. granulomatis, beefy red ulcer

Question: Which vitamin is deficient in children with chronic ascariasis?

Answer: Vitamin A.

Question: List some risk factors for hypothyroidism?

Answer: Women over 60 years old, family or personal history of autoimmune disease, prior thyroid surgery or neck radiation. Rapid Review Hashimoto Thyroiditis Most common cause of hypothyroidism Autoimmune F > M Incidence ↑ with age Risk factor for non-Hodgkin lymphoma

Question: Is computed tomographic colonography used in colorectal cancer screening?

Answer: Yes - it is starting to be used as a screening tool. Rapid Review Colorectal Cancer Screening Routine: Fecal occult blood testing, sigmoidoscopy, or colonoscopy in adults, beginning at 50 until 75 1 relative with colon cancer: colonoscopy at 40 or 10 years earlier than the age at which relative diagnosed Familial adenomatous polyposis: sigmoidoscopy at age 12, then every 1-2 years

Question: At what pH is a parapneumonic effusion or empyema suggested?

Answer: pH below 7.2 Rapid Review Pleural Effusion Transudate: CHF (most common) Exudate: infection > malignancy, PE ↓ Breath sounds + dull percussion + ↓ tactile fremitus CXR: blunting of the costophrenic angle

Smooth, circular, discrete patches of hair loss, characterized by short broken "exclamation point" hairs at the margins, are most consistent with which diagnosis? Alopecia areata Cicatricial alopecia Tinea capitis Trichotillomania

Correct Answer ( A ) Explanation: Alopecia areata is an autoimmune form of nonscarring hair loss. "Exclamation point hairs," which are short broken hairs for which the proximal end of the hair is narrower than the distal end, are pathognomonic for alopecia areata. There are no epidermal changes to the skin during alopecia areata, so the region of hair loss remains smooth. Intralesional injections of corticosteroids are the treatment of choice and most recover from the hair loss within a year. Cicatricial alopecia (B) is an inflammatory form of hair loss, featuring permanent destruction of the hair follicle and loss of the follicular orifice, which leads to a characteristic scaring, unlike alopecia areata. Tinea capitis (C) is a dermatophyte infection of the scalp, usually seen in children, that presents with scaly patches of hair loss, and inflammation, unlike the smooth patches of alopecia areata. Nervous hair pulling or trichotillomania (D), produces a pattern of varying length hairs throughout the patch, described as a "wire brush" feel, rather than a smooth patch with short broken hairs along the margins.

Which of the following signs has the greatest likelihood ratio for acute otitis media? Impaired mobility of the tympanic membrane Red tympanic membrane Retracted tympanic membrane Ruptured tympanic membrane

Correct Answer ( A ) Explanation: A tympanic membrane with distinctly impaired mobility has an adjusted likelihood ratio of 31. A tympanic membrane with slightly impaired mobility has an adjusted likelihood ratio of 4. A tympanic membrane with normal mobility is unlikely to be consistent with acute otitis media. A distinctly red tympanic membrane (B) has an adjusted likelihood ratio of 2. A retracted tympanic membrane (C) has an adjusted likelihood ratio of 0.6. A ruptured tympanic membrane (D) is nonspecific for acute otitis media. Another sign that suggests acute otitis media is a cloudy tympanic membrane with an adjusted likelihood ratio of 34.

A pH of 7.1, HCO3 of 15 and PCO2 of 30 is best described by which of the following primary acid-base disorders? Metabolic acidosis Metabolic alkalosis Respiratory acidosis Respiratory alkalosis

Correct Answer ( A ) Explanation: Acidosis and alkalosis are processes that lead to acidemia (pH < 7.40) and alkalemia (pH > 7.40). Primary metabolic disorders result from a change in bicarbonate, while primary respiratory disorders result from a change in partial pressure of carbon dioxide. Compensation occurs when other system alterations bring the blood gas toward a normal pH of 7.35. A metabolic acidosis is present in any patient with a pH of < 7.35 and bicarbonate < 24. Causes of an increased anion gap acidosis [Na+ - (Cl- + HCO3-)] > 20 can be remembered by the MUDPILES mnemonic (Methanol, Metformin, Uremia, Diabetic (or alcoholic) ketoacidosis, Paraldehyde. Isoniazid, Iron, Lactic acidosis, Ethylene glycol, Salicylates). Normal anion gap acidosis is caused by: renal losses (tubular acidosis, acetazolamide), GI losses (diarrhea, malabsorption), and adrenal insufficiency. Compensation for an acid-base disorder never completely normalizes the pH. A pH of 7.45 in a patient with low bicarbonate indicates a second disorder (such as a primary respiratory alkalosis). Metabolic alkalosis (B) is caused by an increase in bicarbonate leading to a pH > 7.35. This occurs secondary to gastric acid loss from vomiting or NG tube suctioning, diuretic use, and adrenocortical hormone excess. Respiratory acidosis (C) is caused by an increase in the partial pressure of carbon dioxide > 40 leading to a pH < 7.35. This is primarily a result of inadequate ventilation or increased dead space. Causes include head or chest trauma, oversedation, obtundation, neuromuscular disorders, Pickwickian syndrome (obesity-hypoventilation syndrome), and COPD. Respiratory alkalosis (D) is caused by a decrease in the partial pressure of carbon dioxide < 40 leading to a pH > 7.35. In this condition, carbon dioxide ventilation outpaces production.

Which of the following is characteristic of testicular torsion? Associated with a loss of the cremasteric reflex Common complication of cystoscopy Elevating the testicle relieves the pain Increased incidence in patients with a history of sexually transmitted infection

Correct Answer ( A ) Explanation: Although occasionally seen in the newborn male, testicular torsion is an acquired condition seen most commonly in infants, teens, and men <30 years of age. It usually presents with abrupt onset of severe scrotal pain that may wake the patient up from sleep. Associated signs and symptoms include fever, nausea, vomiting, and abdominal pain. On physical examination, the affected testicle is tender and swollen. The cremasteric reflex is invariably absent. The reflex is induced by stroking the inner thigh. The normal response is a contraction of the cremasteric muscle which pulls the ipsilateral testis upwards. The testis may lie in a more horizontal position caused by a lack of normal attachment to the tunica vaginalis ("bell clapper" deformity). If the diagnosis is suspected, immediate consultation with a urologist is warranted. Ultrasound can aid the diagnosis, but can be falsely negative. Timeliness of the diagnosis is critical because testicular viability rapidly declines with time. A history of sexually transmitted infections (D) does not increase the likelihood for testicular torsion. Epididymitis is a complication of cystoscopy (B), not torsion. Elevating the testicle (C) usually induces pain, rather than relieves pain. In some cases, testicular elevation relieves pain in epididymitis (Prehn's sign).

18 months ago, an elderly patient received a mitral valve replacement. Unfortunately, for the past year, he has been fighting subacute bacterial endocarditis. He has been admitted to the hospital 3 times in the past 6 months. Of all things, he is most concerned with unsightly changes of his palms. During inspection, you appreciate that both palms have several nontender macules of red to brown to black coloration. His daughter is getting married in 3 weeks, and he doesn't want people to see these "weird rashes" on his hands. Which of the following correctly names these findings? Janeway lesions Raynaud's phenomenon Roth spots Splinter hemorrhages

Correct Answer ( A ) Explanation: Bacterial endocarditis usually presents with persistent fever, bacteremia and constitutional symptoms. There are also several physical examination findings which clue the clinician into considering this diagnosis. A new onset murmur, a change in a previous murmur or a new thrill are common presenting signs. Conjunctival and palate petechiae are also common. Tender splenomegaly or tender joints may be present. Neurologic symptoms may be the result of septic emboli emanating from an acutely infected cardiac valve. Septic emboli can also present with cutaneous findings, namely Janeway lesions, however, these are usually present in subacute rather than acute endocarditis. They are described as multicolored hemorrhagic macules which appear on the palms and soles. Treatment of prosthetic valve endocarditis of >1 year's duration is vancomycin plus gentamicin plus ceftriaxone, adjusted, of course, to culture results. Raynaud's phenomenon (B) commonly occurs in rheumatologic disease, like systemic sclerosis. It represents a cold-induced distal digital vasoconstriction, which causes finger, not palmar, pallor, cyanosis or hyperemia and pain. Roth spots (C), also a common finding in bacterial endocarditis, represent retinal hemorrhages surrounding pale centers. They do not occur on the palms. Other cutaneous findings of bacterial endocarditis include proximal nail bed, not palmar, splinter hemorrhages (D).

A 57-year-old woman with a long history of poorly-controlled diabetes mellitus presents with worsening edema in her legs and an occasional "wet-sounding" cough. Physical exam shows 2+ pitting edema extending to mid-calf bilaterally and trace periorbital edema. Blood tests show hypoalbuminemia and hyperlipidemia and urinary protein excretion is 3.5 grams/24 hours. Which of these complications may occur as a direct result of her new condition? Deep vein thrombosis Gouty arthritis Peripheral neuropathy Struvite calculi

Correct Answer ( A ) Explanation: Deep vein thrombi secondary to a hypercoagulable state may occur in patients with nephrotic syndrome due to excessive urinary protein losses. As serum albumin levels drop below 2 grams/dL, patients typically become deficient in antithrombin, protein C, and protein S, which causes an increased propensity to clotting. Patients with nephrotic syndrome who develops a thrombosis in any location should be anticoagulated for 3-6 months; recurrent thrombi may warrant anticoagulation indefinitely. Nephrotic syndrome is characterized by the triad of hypoalbuminemia, hyperlipidemia, and proteinuria greater than 3 grams/24 hours. Symptoms include dependent edema that can become generalized, ascites, and pulmonary edema. One third of adults with nephrotic syndrome have an underlying systemic disease such as diabetes mellitus, systemic lupus erythematous, or amyloidosis, as opposed to one of many primary renal diseases. Nephrotic syndrome is diagnosed based on the above blood and urinalysis findings. Renal biopsy is typically needed to determine the etiology unless there is a known likely secondary cause. Treatment requires increasing the patient's daily protein intake and using an ACE-inhibitor or angiotensin receptor-blocker to reduce protein loss. Salt restriction and thiazide or loop diuretics aid in edema management. Statins and diet changes may help manage the hyperlipidemia. Gouty arthritis (B) causes acute joint point and is related to overproduction of uric acid or underexcretion of uric acid by the kidney. Neither etiology is directly related to nephrotic syndrome. Peripheral neuropathy (C) may occur in this patient secondary to her poorly controlled diabetes but it is not caused by nephrotic syndrome. Struvite calculi (D) are referred to as "infection stones" as they typically occur in patients with recurrent urinary tract infections with urease-splitting organisms. These stones generally do not occur as a result of nephrotic syndrome.

26-year old sexually active man presents with a 3-day history of unilateral, painful testicular swelling. He reports subjective fevers and dysuria and denies nausea and vomiting. Urinalysis shows leukocyte esterase and greater than 10 white blood cells. What is the next step in management for this patient? Doxycycline and ceftriaxone therapy Levofloxacin Testicular ultrasound Urethral swab and Gram stain

Correct Answer ( A ) Explanation: Epididymitis is the most common cause of scrotal pain in adults and is characterized by acute unilateral pain and swelling. The pain usually begins at the epididymis and can spread to the entire testicle (epididymo-orchitis). Other symptoms include fever, erythema of the scrotal skin, and dysuria. It is associated with a C-reactive protein level greater than 24 mg per L and increased blood flow on ultrasonography. Chlamydia trachomatis and Neisseria gonorrhoeae are the most common organisms responsible for bacterial epididymitis in males younger than 35 years. Guidelines recommend ceftriaxone (250 mg IM x 1) and doxycycline (100 mg PO BID x 10 days) for treatment of suspected epididymitis in males younger than 35 years. Levofloxacin (B) is the treatment of choice for epididymitis-orchitis in males older than 35 years of age. The most common pathogens in this population are the enteric bacteria (e.g. E. coli). A testicular ultrasound (C) is indicated if there is any concern for testicular torsion, a surgical emergency that requires immediate consultation with a urologist. Characteristics that suggest testicular torsion include rapid symptom onset, nausea and vomiting, high position of the testicle, and absent cremasteric reflex. Surgical repair must occur within six hours of symptom onset to reliably salvage the testicle. Urethral swab and Gram stain (D) is typically reserved for patients with suspected acute urethritis and not epididymitis.

A patient with rheumatoid arthritis presents with new onset generalized weakness, fatigue, facial swelling and weight gain. Examination is significant for periorbital edema, dry skin and coarse brittle hair. Laboratory testing reveals a high titer of antithyroid antibodies. Which of the following will most likely be prescribed? Levothyroxine Methimazole Metoprolol Radioactive iodine

Correct Answer ( A ) Explanation: Hypothyroidism is most commonly caused by Hashimoto's thyroiditis. This autoimmune disease occurs with other autoimmune disease, like rheumatoid arthritis, and is marked by the presence of antibodies that attack thyroid cells, specifically antithyroid peroxidase and antithyroglobulin autoantibodies. Patients may present transiently with signs of hyperthyroidism due to inflammation of the thyroid, however, hypothyroidism shortly prevails. Primary (non-central) hypothyroidism is heralded by a low free-T4 and a high TSH. Elevated TSH in the presence of a normal T4 is considered subclinical hypothyroidism. Hypothyroidism causes a generalized slowing of metabolic processes, leading to a vague, nonspecific constellation of symptoms that include fatigue, weight gain, cold intolerance, dry skin, constipation, menstrual irregularities, and depression. Treatment is only necessary in symptomatic patients by thyroid hormone replacement. This is accomplished by starting the synthetic thyroid hormone levothyroxine, monitoring TSH levels every 6 weeks, and titrating until symptoms resolve and the lab abnormalities normalize. Increased dosing is commonly needed during pregnancy. Lower starting doses are used in those with ischemic heart disease. Methimazole (B), metoprolol (C) and radioactive iodine (D) are all used for the treatment of hyperthyroidism, not hypothyroidism. Methimazole blocks enzymes responsible for thyroid hormone biosynthesis. Metoprolol's beta-blocking action treats the adrenergic symptoms, namely tachycardia. RAI is used as ablative therapy for those with hyperthyroidism.

A 14-year old girl is accompanied by her parents for amenorrhea. She never had menses. She denies sexual activity and does not take any medications. She is a straight A student in school and plays basketball. On physical examination, her height is at the 90th percentile, weight at the 50th percentile, breasts at Tanner II, no axillary hair and no pubic hair. The rest of her examination is normal. Which of the following is the most likely diagnosis? Complete androgen insensitivity syndrome Klinefelter syndrome Polycystic ovarian syndrome Turner syndrome

Correct Answer ( A ) Explanation: The girl in the vignette most likely has complete androgen insensitivity syndrome (AIS) (formerly known as testicular femininization). Complete AIS is an extreme form of failure of virilization wherein genetic males appear female at birth and are invariably reared accordingly. The external genitalia are female. The vagina ends blindly in a pouch, and the uterus is absent. This is due to the normal production and effect of antimullerian hormone by the testes. The testes are usually intra-abdominal although some may descend into the inguinal canal. At puberty, the diagnosis is suspected by the normal development of breasts, but menstruation does not occur and sexual hair is absent. Adult heights are commensurate with those of normal males. Management of patients with complete AIS involves removal of testes whose sexual orientation is unambiguously female. This can be done through laparoscopic removal of Y chromosome-bearing gonads. Klinefelter syndrome (B) has XXY chromosomes and presents with gynecomastia, small testicles, infertility, mild delays, and a low upper-to-lower segment ratio. Polycystic ovarian syndrome (C) presents with irregular menses, acne, excess body hair, and signs of insulin resistance. Turner syndrome (D) has XO chromosomes and presents with breast buds, short stature, pedal edema, wide-spaced nipples. They may have primary amenorrhea from streak ovaries and ovarian failure.

A 65-year-old obese man presents to your office with complaints of snoring and fatigue. His wife makes him sleep in a separate room because of his loud snoring and gasping for air in his sleep. Which of the following is the most appropriate therapy? Continuous positive airway pressure therapy Modafinil Oxygen therapy Theophylline

Correct Answer ( A ) Explanation: Obstructive sleep apnea (OSA) is a disorder characterized by a decrease or cessation of airflow during sleep. It is more common in men. The prevalence increases with age and obesity is the most well-documented risk factor. The classic symptoms of OSA are snoring, spousal report of apnea, daytime sleepiness, fatigue and hypoxia. Diagnosis of OSA is confirmed by a sleep study. All patients with OSA should be counseled on behavioral modifications, including weight loss, sleeping in the lateral recumbent position, smoking cessation, and avoiding alcohol for 4-6 hours prior to bedtime. The cornerstone of OSA treatment is continuous positive airway pressure (CPAP) therapy. Over time, untreated OSA may lead to increased morbidity and mortality due to hypertension, heart disease, stroke, diabetes or metabolic syndrome. Modafinil (B) is a central nervous system stimulant sometimes used as adjunctive treatment of the daytime sleepiness symptoms associated with OSA. Oxygen therapy (C) may be used as an alternative in patients who are non-compliant or unable to tolerate CPAP therapy. Theophylline (D) is a phosphodiesterase enzyme inhibitor thought to stimulate the respiratory drive, however pharmacologic therapy is not effective enough to replace CPAP as first-line therapy.

Which of the following is true regarding older patients with abdominal pain in comparison to younger patients? More likely to need an emergent surgical procedure More likely to present with an elevated white blood cell count More likely to present with fever More likely to present with peritoneal signs

Correct Answer ( A ) Explanation: Older patients with abdominal pain are more likely to present atypically and more likely to present with life-threatening causes requiring emergent surgery. Abdominal pain can be a diagnostic challenge in all patients due to the variability in presentation of common intraabdominal pathologies. Often, an exact etiology is not found on initial evaluation. However, the likelihood of having a benign cause decreases with increasing age. Older patients with abdominal pain are more likely to harbor serious pathology but will often present atypically and progress rapidly. The decrease in diagnostic accuracy and increased rate of severe disease results in an increased mortality for older patients. In those older patients where specific causes are found, they are more likely to require a surgical procedure. Fever (C), elevated WBC counts (B) and peritoneal pain (D) are less common in older patients in comparison to younger patients.

You diagnose strabismus in a 6-month-old girl. During your discussion with the parents, you educate them on the typical surgical procedure. Which of the following would most likely be included in your discussion? Extraoccular muscle lengthening Laser correction of corneal defects Laser-incision and drainage through the iris Ultrasound vibration of the lens

Correct Answer ( A ) Explanation: Strabismus is a condition of globe misalignment that results in the inability of both eyes to fixate on the same point in space, diplopia, poor depth perception and decreased binocular vision. Goals of treatment include obtaining near normal acuity, obtaining or improving fusion, eliminating any associated sensorimotor adaptations (as in adaptive compensatory head-positioning) and obtaining a favorable functional appearance of normal eye alignment. Interpersonal eye contact and positive self-image rely heavily on the normal position of the eyes, as such, treating strabismus is important for acuity and psychological matters. Treatment options include refractive correction, use of prisms, vision therapy, chemodenervation, pharmacologics and extraoccular muscle surgery. Strabismus surgery is based on lengthening or shortening the extraoccular muscles, repositioning them on the globe so as to achieve normal alignment. LASIK (B) uses laser or microtome correction for refractive disorders, not strabismus. Laser (C) treatment is used to allow drainage through the iris in the treatment of acute closed-angle glaucoma. Phacoemulsification, the most common treatment of cataract, uses ultrasound vibrations (D) to dissolve a clouded lens.

A 30-month-old male who was recently adopted from China presents to the Emergency Center after the rapid onset of difficulty breathing. His temperatures is 103°F. He preferentially sits leaning on his arms, which are outstretched in front of him, and he is drooling. What is the most likely etiology of his respiratory distress? Haemophilus influenzae type B Parainfluenza virus Respiratory syncytial virus Staphylococcus aureus

Correct Answer ( A ) Explanation: The child presents with symptoms of epiglottitis, a life-threatening upper airway obstruction that requires immediate recognition and careful airway management. Symptoms present abruptly and include respiratory distress, drooling, dysphagia, and a high fever. Affected children are toxic-appearing and often assume the "tripod" position to maximize upper airway patency. Although epiglottitis is rare in the era of routine vaccination, Haemophilus influenzae type B remains the most common cause amongst both vaccinated and unvaccinated children. The above child should raise suspicion for incomplete vaccination because of his recent adoption from abroad. The first step in management of epiglottitis is to secure an airway. Every effort should be made to keep the child calm, and he should be taken to the operating room or intensive care unit for emergent intubation for airway protection. Empirical antibiotic therapy should include coverage of H.influenzae, as well as S.aureus, S.pyogenes, and S.pneumoniae. Parainfluenza virus (B) and Staphylococcus aureus (D) are also recognized causes of epiglottitis but are not as common as H.influenzae, even in the era of routine H.influenzae vaccination. Respiratory syncytial virus (C) is not a common cause of epiglottitis but is the common cause of bronchiolitis, a common cause of lower airway obstruction in children less than two years of age. Bronchiolitis classically presents with two to three days of nasal congestion and cough, which may progress to increased work of breathing. High fevers, drooling, and dysphagia are not characteristic of bronchiolitis.

What medication should be first line treatment in moderate musculoskeletal back pain? Acetaminophen Cyclobenzaprine Diazepam Oxycodone

Correct Answer ( A ) Explanation: The first choice for patients with mild to moderate back pain should be acetaminophen or a nonsteroidal anti-inflammatory drug (NSAID) like ibuprofen or naprosyn. In patients with severe pain, opiate medications may be necessary. However, it is important to note that no studies have demonstrated the superiority of one pain medication (or class of medication) over another in the treatment of back pain. Patients with milder symptoms should initially be treated with acetaminophen or an NSAID. These medications are well tolerated in short courses with minor side effects. NSAIDs have not been shown to be superior to acetaminophen. Cyclobenzaprine (B) and diazepam (C) are muscle relaxants that promote sleep but have no evidence supporting their effectiveness in the treatment of back pain. Oxycodone (D) is an opiate analgesic with significant side effects and no proven benefit over more conservative treatments.

A four-week-old baby boy presents to the Emergency Department with projectile vomiting over the past three days. On physical exam, he is found to have a palpable olive-like mass in the right upper quadrant. On laboratory analysis he has a hypochloremic metabolic alkalosis. Which of the following is a risk factor for development of this condition? Erythromycin use Female sex Full-term birth Second born child

Correct Answer ( A ) Explanation: The infant has pyloric stenosis, and erythromycin use is a risk factor. Pyloric stenosis occurs when there is hypertrophy of the pylorus, which causes a gastric outlet obstruction. It is most commonly seen between three to five weeks after birth. Risk factors include first born, male sex, prematurity, and macrolide antibiotic use. Infants will present as a "hungry vomiter" with forceful nonbilious vomiting with a desire to feed again. On physical exam, a palpable olive may be found in the right upper quadrant. Repetitive vomiting will lead to a hypochloremic metabolic alkalosis from loss of stomach acid. If this continues for a prolonged period, hypokalemia will occur due to stimulation of aldosterone from hypovolemia. Abdominal ultrasound will show a "target sign" that is characterized by an increased pyloric muscle thickness, length, and diameter. Definitive treatment is surgical management with a pyloromyotomy. Male sex, not female sex (B) is a risk factor for pyloric stenosis. Similarly, full-term birth (C) and being second born (D) are not risk factors.

A 53-year-old man presents with a swollen, painful left knee. He denies trauma or fever. Examination reveals decreased range of motion of the left knee with swelling and warmth. What test is likely to be diagnostic? Arthrocentesis and fluid analysis Serum uric acid level Serum white blood cell count X-ray of the knee

Correct Answer ( A ) Explanation: The patient presents with atraumatic, monoarticular arthritis with warmth and swelling and must have an arthrocentesis to rule out a septic joint. Patients with septic arthritis will typically present with fever, joint pain, swelling and effusion in a single joint. Additionally, patients frequently have warmth of the joint and decreased range of motion. Involvement of multiple joints raises the possibility of meningococcal disease and sepsis. Most septic joints are caused by gram-positive bacteria (75-90%) with Staphylococcus aureus being the most common. Although there are a number of serum lab tests that are often requested in the workup of a possible septic joint (WBC, ESR, CRP) systematic reviews show little evidence that these tests can rule in or out the disease. The only way to obtain a definitive diagnosis is with arthrocentesis and fluid analysis. A synovial fluid WBC count is traditionally considered the most useful test. A WBC >50,000 cells is highly suggestive of a septic join. Synovial lactate level >5.6 mmol/L has been found to be highly predictive of a septic joint, as well. Treatment of a septic arthritis involves antibiotics and orthopedic consultation for possible open drainage. A serum uric acid level (B) is helpful in the diagnosis of gouty arthritis but not in septic arthritis. Serum WBC count (C) is neither sensitive nor specific for this diagnosis. Radiographs (D) are often performed and may shows effusion or possibly osteomyelitis but are not diagnostic.

The parents of a 9-month-old boy are concerned about their newborn's foreskin. They are not considering circumcision, but are worried about the immobility of the foreskin, stating that they cannot fully retract it over the glans penis. You explain to them that a nonretractbale foreskin, if not associated with symptoms, is considered normal up to which of the following age groups? Adolescence (13-22 years old) Adulthood (>23 years old) Childhood (4-12 years old) Infancy (<4 years old)

Correct Answer ( A ) Explanation: There are two conditions to consider when discussing foreskin abnormalities: phimosis and paraphimosis. Phimosis is a condition in which the foreskin cannot be fully retracted over the glans penis. At birth, the foreskin is normally nonretractable, and may remain as such adolescence (retractable in 50% of 1-year-olds, 90% of 3-year-olds and 99% of 17-year-olds). If the infant, child or adolescent is asymptomatic, then this is considered physiologic phimosis. On the other hand, pathologic phimosis, although rare, refers to a nonretractable foreskin that causes problems such as penis inflammation with possible ischemia (paraphimosis), difficulties with urination or difficulties with sexual function. Underlying causes and associations include lichen sclerosis, balanitis, inappropriate, forcible or prolonged foreskin retraction, repeated urinary catheterization, unusual masturbation techniques and a short frenulum. When symptoms arise and are not severe, conservative measures may be appropriate. These include topical steroid creams and manual or device-assisted tissue expansion. If symptoms are severe, or cause acute complications, surgical intervention may be required, and include circumcision, dorsal or ventral slit incisions or preputioplasty. Phimosis in adulthood (> 23 years old) (B) is considered pathologic. Phimosis can be physiologic past infancy (< 4 years old) (D) and childhood (4-12 years old) (C) and into adolescence.

A 19-year-old man presents to the ED with pain along his penile shaft for the past 7 days. He reports a low-grade fever and myalgias and was sexually active with a new partner 10 days ago. He denies any penile discharge or dysuria. There is no inguinal adenopathy palpated on exam, but he has tender penile lesions, revealed in the image seen above. Which of the following is most likely to be an effective treatment? Acyclovir PO Ceftriaxone IM Doxycycline PO Penicillin G IM

Correct Answer ( A ) Explanation: This is a patient with a primary herpes infection, characterized by a low-grade fever; myalgias; and multiple painful, shallow, tender, genital lesions. These typically follow a 2-7-day incubation period. Patients often do not have adenopathy until the 2nd or 3rd week of illness. Lesions last for 2-4 weeks. Treatment is acyclovir to reduce the duration of symptoms and viral shedding. Ceftriaxone (B) is one of several treatment regimens for chancroid. Although chancroid also presents with painful genital ulcers, 50% of patients also develop a large unilateral, fluctuant lymph node (bubo) 1 week after the ulcers appear. It is often difficult to differentiate herpes from chancroid on exam. However, herpes is orders of magnitude more common in the United States; most cases of chancroid occur in developing countries. There are usually fewer than 100 cases of chancroid reported to the CDC annually. Therefore, in patients such as this one, herpes should be of primary consideration, and acyclovir is more likely to be an effective treatment. Doxycycline (C) is used to treat lymphogranuloma venereum. However, lesions are painless and often go unnoticed. Patients often present in the secondary stage following the disappearance of genital lesions, when they develop painful lymphadenitis. Penicillin (D) would be the appropriate treatment for syphilis. However, the incubation period is typically longer and primary lesions (chancres) are painless with raised edges.

A 7-year-old boy accompanied by his mother presents to your office, complaining of a rash and bilateral knee and ankle pain. He also notes some mild crampy abdominal pain associated with nausea but denies vomiting, diarrhea, and fever. On exam you note the rash seen above. He has swelling around his patellar joints bilaterally, and the area is very painful. There are no palpable abdominal masses. His lungs are clear and the remainder of the physical examination is unremarkable. What complication is this patient at greatest risk for? Intussusception Joint damage Pneumonia Thrombocytopenia

Correct Answer ( A ) Explanation: This patient has Henoch-Schönlein Purpura (HSP). Henoch-Schönlein Purpura is the most common systemic vasculitis of childhood and is characterized by a rash (palpable purpura), arthritis, abdominal pain, and renal involvement. The exact etiology is unknown, but it is often associated with an antecedent bacterial or viral infection. It affects boys more commonly than girls and the median age is 5-years-old. The most striking feature of HSP is the characteristic rash of palpable purpura that typically occurs on the lower extremities and buttocks and is present in all cases of HSP. The rash may blanch with pressure. Gastrointestinal involvement is typically seen during the acute phase of illness and may precede the rash. It is thought to be due to small vessel involvement and usually presents as gastrointestinal colic but may lead to ischemia (heme positive stools) and intussusception. When intussusception occurs, it is usually ileoileal, unlike the more common ileocolic, and is more difficult to diagnose and reduce by barium or air contrast enema. Renal involvement will usually present early and can range from microscopic hematuria to nephrotic syndrome. Additionally, up to 15% of males can have scrotal involvement with epididymitis, orchitis, torsion, and scrotal bleeding. The musculoskeletal involvement is periarticular and does not involve the actual joint. It tends to affect the lower extremities, especially the ankles and knees. It can be very painful but does not leave any permanent joint damage. Treatment is supportive. The purpura is nonthrombocytopenic, and thrombocytopenia (D) is not seen in HSP. The platelet count should be normal. Pneumonia (C) is not a known complication of HSP. Joint damage (B) does not occur as HSP is associated with periarticular disease.

In a patient with suspected HIV infection and shortness of breath, which of the following findings is most suggestive of Pneumocystis jiroveci pneumonia? Bradycardia despite relative volume depletion Elevated serum lactate dehydrogenase Hyponatremia Unilateral lobar consolidation on chest radiograph

Correct Answer ( B ) Explanation: Pneumocystis pneumonia is one of the most common opportunistic infections in patients with HIV. Patients typically present with an insidious nonproductive cough, dyspnea, unexplained fever for longer than two weeks, chest pain, and fatigue. The greater the elevation of lactate dehydrogenase (LDH), the worse the prognosis. Relative bradycardia (A) is commonly associated with typhoid fever (Salmonella typhi), Legionnaire's disease (Legionella pneumophila), infectious mononucleosis, and pneumonia caused by Chlamydia species. The syndrome of inappropriate antidiuretic hormone secretion (SIADH) is one of several causes of hyponatremia (C). SIADH may be seen in patients with certain pulmonary infections, including Legionnaire's disease and tuberculosis, though the mechanism is poorly understood. SIADH has not been reported with Pneumocystis jiroveci pneumonia. The chest radiograph (D) in Pneumocystis jiroveci pneumonia commonly shows a diffuse interstitial infiltrate but may also be normal in appearance or reveal asymmetrical nodules, cavitation, or bullae.

You are the primary care provider for a 49-year-old man with hypertension and diabetes mellitus. He turns 50 years old in two weeks. You discuss upcoming colorectal cancer screening. According to the 2008 US Preventive Services Task Force, you make which of the following recommendations? Colonoscopy done every 5 years Fecal occult blood testing done every year Rectal biopsy done once after turning 50 years of age Sigmoidoscopy done every 10 years

Correct Answer ( B ) Explanation: According to the 2008 recommendations, the benefits of colorectal cancer screening outweigh the potential harms for 50-75 year olds. There is no recommendation for screening for patients greater than 76 years of age, citing the likelihood that detection and early intervention will yield a mortality benefit declines after age 75 because of the long average time between adenoma development and cancer diagnosis. Screening is accomplished with high sensitivity fecal occult blood tests, sigmoidoscopy or colonoscopy. The intervals are as follows: fecal occult blood test every year, or fecal occult blood test every 3 years PLUS sigmoidoscopy every 5 years (D), or colonoscopy every 10 years (A). Routine screening biopsy (C) of the rectal mucosal wall is not included in these recommendations.

A 5-year old boy with a history of anaphylaxis to penicillin presents with exudative pharyngitis and a positive rapid antigen test for Group A Strep. What is the treatment of choice? Cefdinir Clindamycin Doxycycline Trimethoprim-sulfamethoxazole

Correct Answer ( B ) Explanation: Clindamycin provides excellent coverage for Group A Strep infections in children with immediate-type hypersensitivity reactions to beta-lactam antibiotics. The risk of resistance of Group A Strep to clindamycin is < 1%. Notably, clindamycin solution has a very harsh taste, so pills are always preferred for children who are able to swallow pills. An alternative to clindamycin is azithromycin. Cephalosporins such as cefdinir (A) are good alternatives for treatment of Group A Strep infections in patients with non-immediate hypersensitivity reactions to penicillin. However, cephalosporins are contraindicated in those with anaphylactic hypersensitivity to penicillin due to a risk of cross-reactivity between the antibiotic classes. Neither doxycycline (C) nor trimethoprim-sulfamethoxazole (D) provide adequate coverage for Group A Strep. Moreover, tetracyclines are not recommended in children under eight years of age due to the risk of dental enamel hypoplasia and staining.

You are evaluating a mentally challenged 45-year-old man for groin pain. On examination you notice that he has large ears, a prominent jaw, and large symmetric testicles. These findings are consistent with which one of the following? Asperger's syndrome Fragile X syndrome Klinefelter's syndrome Variant form of Down syndrome

Correct Answer ( B ) Explanation: Fragile X syndrome accounts for more cases of mental retardation in males than any other genetic disorder except Down syndrome. Fragile X syndrome is caused by an expansion mutation in the Fragile X mental retardation 1 (FMR1) gene located on the X chromosome. It characteristically leads to some degree of mental retardation. The phenotype is subtle, with minor dysmorphic features and developmental delay during childhood. Characteristic features during adolescence are an elongated face, prominent jaw, large ears, macro-orchidism, and a range of behavioral anomalies and cognitive deficits. Behavior is characterized by attention deficits, hand flapping, hand biting, and gaze aversion. The diagnosis of fragile X syndrome is confirmed by molecular genetic testing of the FMR1 gene. Asperger's syndrome (A) is a variant of autism in people of normal to high intelligence. Physical findings are insignificant however delays in social interactions, and restricted and repetitive behaviors are characteristics of this disorder. Down syndrome (D) is a chromosomal abnormality known as trisomy 21 and has a collection of clinical features which include micrognathia, macroglossia, epicanthal folds, flattened facial appearance, single transverse palmar crease as well as many other distinguishing features. This patient does not have any features of Down syndrome. Klinefelter syndrome (C) is caused by an additional X chromosome in males (47, XXY). Clinical findings are nonspecific during childhood; thus, the diagnosis commonly is made during adolescence or adulthood in males who have small testes with hypergonadotropic hypogonadism and gynecomastia. Virtually all men with Klinefelter syndrome are infertile.

A 4-month-old infant presents with erythematous plaques to the bilateral cheeks that have been worsening over the past three weeks. Similar plaques are noted over both elbow extensors. He has been growing and developing normally. What is the treatment of choice? Reassurance Topical emollients Topical mupirocin Topical triamcinolone

Correct Answer ( B ) Explanation: The infant has atopic dermatitis, also known as eczema. In infants, eczema is most common on the cheeks and extensor surfaces of the extremities. This distribution is in marked contrast to in children and adolescents who classically have involvement of the flexor surfaces, such as the antecubital and patellar fossas. The first-line treatment for eczema is a topical emollient. Thick, unscented creams and ointments and preferred to lotions. Emollients should be applied at least twice per day and re-applied after bathing, swimming, or hand-washing. If the eczema is persistent, a topical steroid may be applied in addition to topical emollients. The lowest effective potency of corticosteroid should be prescribed, and steroids should be applied only to affected skin to avoid the side effects of skin thinning, atrophy, and telangectasias. Desonide or hydrocortisone are good initial low-potency topical corticosteroids. Reassurance (A) alone is not an appropriate treatment for eczema, as uncontrolled eczema may lead to significant itching and excoriations, which creates microtears that predisposes children to superinfection. Additionally, eczema may lead to hyperpigmentation and scarring. Topical mupirocin (C) is a topical antibiotic that is indicated for impetigo or mild, localized superinfections of eczema. It is not indicated for routine care of eczema. Topical triamcinolone (D) is a mid-potency corticosteroid that may be used to treat eczema that does not respond to topical emollients and low-potency corticosteroids. It should not be used on the face or genital region and should only be applied to affected skin.

A 6-year-old boy presents to your office because his teachers noticed that he frequently tends to stare off during some of his schoolwork and will not respond to their questions. His mother states that she has seen him stare off during dinner but assumed that he was just ignoring her. What is your next step in the evaluation of this child? Order a MRI Order an EEG Reassure the mother and tell her it is likely nothing Send him to the ER for urgent treatment

Correct Answer ( B ) Explanation: This patient is having absence seizures (petit mal) and requires an EEG. Absence seizures are characterized by sudden-onset staring spells, usually lasting approximately 10 seconds, with immediate recovery. Clinically, patients may experience eye fluttering or altered postural tone that may occur many times per day. The diagnosis is confirmed by EEG, which shows characteristic generalized 3 Hz spike-and-wave discharges. Childhood absence epilepsy is an idiopathic generalized epilepsy that has an onset between 4 and 14 years of age with a peak at 5 to 7 years. The prognosis is generally favorable with approximately 80% of children outgrowing their seizures by adolescence. An MRI (A) is warranted if there is a concern for a structural brain lesion (tumor). However, this patient's presentation is classic for absence seizure and he does not require an MRI at this time. Mom should be reassured (C) but not told, "it is likely nothing," since the patient has absence seizures and will require medical therapy. Absence seizures are typically not life threatening and do not require urgent evaluation in the ER (D). The patient will require referral to a pediatric neurologist.

A man who presents with syncope is placed on the cardiac monitor. On the monitor you note a repeating trend of 6 P waves, 5 of which are followed by a narrow QRS complex and 1 of which is not followed by a QRS complex. The PR interval during this trend progressively increases. Which of the following is the most likely diagnosis? First-degree AV block Third-degree AV block Type I second-degree AV block Type II second-degree AV block

Correct Answer ( C ) Explanation: A key distinction between first-degree and second-degree heart block is that in first-degree block the P wave is always followed by a QRS complex. In other words, the ratio of P waves to QRS complexes is 1:1, or, the electrical signal from the atria always passes to the ventricles. In second-degree AV block, the electrical impulse sometimes gets to the ventricles. There are two main types of second-degree AV block. In Mobitz type I, or Wenckebach, second-degree block, there is a progressive beat-to-beat lengthening of the PR interval until a P wave does not conduct through the AV node. The absent conduction and resultant "missing" QRS complex is called a "dropped" QRS, which represents an absent beat of ventricular contraction. First-degree AV block (A) has a 1:1 ratio of P waves to QRS complexes. Mobitz type II second-degree heart block (D) is characterized by a nonconducted P wave which is not preceded by progressive PR interval prolongation. AV dissociation, or third-degree AV block (B), occurs when none of the P waves conduct through the AV node. This complete AV block occurs with separate atrial and ventricular rates. There is no discrete correlation or trend between P waves and QRS complexes.

A businessman, who frequently travels in airplanes, complains of painful ear popping every time he flies. He has tried yawning, swallowing, chewing gum and pinching his nose, but nothing seems to relieve the pain. He has even tried using ibuprofen two hours prior to flight. Which of the following would you recommend? Myringotomy Nifedipine Oxymetazoline Ranitidine

Correct Answer ( C ) Explanation: Barotitis media, barotrauma, or ear popping, are all names for eustachian tube dysfunction, a condition that occurs when the tube does not open properly during swallowing or yawning. This tube, connecting the middle ear to the pharynx, equalizes pressure inside the ear to the atmospheric pressure. Blockage of the tube can arise from congenital stenosis, abnormal peristaltic function, abnormal ciliary function, adenoid hypertrophy, nasal congestion and tumors. Risk factors include ear or sinus infections, allergies and rapid altitude changes. Symptoms include ear fullness, otalgia, tinnitus, hearing impairment and vertigo. Diagnosis is mainly clinical, but further evaluation may involve an audiogram, tympanogam or otolaryngologist referral. Treatment includes chewing gum, multiple swallows and exhalation through closed nostrils. If refractory, medications may be necessary, and include analgesics, oral antihistamines and nasal decongestants or steroids. Oxymetazoline is an over-the-counter nasal spray decongestant. Rare cases require a myringotomy (A) to alleviate severe acute symptoms. This patient has not tried oral or nasal medications yet. Nifedipine (B), a calcium-channel blocker, is used to treat hypertension, not eustachian tube dysfunction. Ranitidine (D) is a histamine-2 antagonist used in the treatment of gastrointestinal ulcer and reflux disease, not nasal congestion.

What is considered the first-line therapy for bulimia? Antidepressants Antipsychotics Cognitive behavioral therapy Weight management plan

Correct Answer ( C ) Explanation: Cognitive-behavioral therapy (CBT) is the best-evidenced approach for treatment of bulimia nervosa. Bulimia nervosa is characterized by binge eating episodes, followed by inappropriate compensatory behaviors, such as self-induced vomiting, laxative or diuretic misuse, fasting, and excessive exercise. A sense of loss of control about overeating accompanies these episodes of binge eating. These behaviors are associated with overvaluation of shape and weight. The episodes must occur at a frequency of two times per week for 3 months to meet diagnostic thresholds. Bulimic behaviors usually have their onset during middle adolescence (14-16 years old). Full-syndrome bulimia nervosa is most common during late adolescence and young adulthood (17-24 years old). Onset of bulimia nervosa is rare in younger children, although not unknown. It is common for other psychiatric disorders to coexist with bulimia nervosa, particularly depression, anxiety disorders, and substance use. Individuals with bulimia have normal or near-normal body weight. Antidepressants (A) are useful but are not as effective as cognitive behavioral therapy and are a second-line treatment. However these medications may be used to augment psychological treatments or as an alternative when psychological treatments are not effective, are refused, or are not available. Antipsychotics (B) are not indicated in the treatment of bulimia. Dysfunctional attitudes toward body shape and weight are the maintaining factors for the bulimia nervosa not the need for a weight management plan (D). As a result of these distorted ideas, there is an overvaluing of appearance, particularly thinness.

Which of the following congenital disorders results in a cyanotic child? Atrial septal defect Coarctation of the aorta Tetralogy of Fallot Ventricular septal defect

Correct Answer ( C ) Explanation: Cyanotic heart disease results from either right-to-left shunting of desaturated blood flowing into the systemic circulation or from decreased pulmonary blood flow. Tetralogy of Fallot is the classic CHD but this group also includes truncus arteriosus, dextroposition (or transposition) of the great arteries, tricuspid atresia and total anomalous pulmonary venous return. The majority of these disorders are identified when patients are still in the nursery.The abnormalities seen in Tetralogy of Fallot include 1) right ventricular outflow tract obstruction, 2) over-riding aorta, 3) ventricular septal defect (VSD) and 4) right ventricular hypertrophy (RVH). An easy mnemonic to remember this is "PROVe" (Pulmonary stenosis, RVH, Overriding aorta, and VSD). Patients typically present with cyanosis that is worse during feeding and crying. They often have poor feeding and in older children, they may squat to relieve symptoms. Patients may experience "tet spells" characterized by acute respiratory distress due to increased right outflow tract obstruction. Patients with severe right ventricular outflow obstruction may present early after birth with profound cyanosis requiring reopening of the patent ductous arteriosus. Patients will have a systolic ejection murmur along the left sternal border. Chest radiography typically shows the classic "boot-shaped" heart. Cyanosis can be recreated by any valsalva maneuver as this results in an increase in the right-to-left shunt across the VSD. Definitive treatment is surgical correction. Atrial septal defect (A), coarctation of aorta (B) and ventricular septal defect (D) are all non-cyanotic lesions.

A 58-year-old man presents with shortness of breath for 2 days. He complains of a six-month history of a dry cough, unintentional weight loss, and night sweats. He has no past medical history, but he has a 40 pack-year smoking history. In the ED, his vital signs are BP 132/76, HR 72, RR 16, oxygen saturation 96% on room air, and temperature 98.8°F. An ECG reveals no acute abnormality, and a chest X-ray shows a right middle lobe irregular mass and a right-sided pleural effusion. A thoracentesis is performed. What findings would be expected on pleural fluid analysis? Fluid:Blood LDH ratio <0.6 Fluid:Blood Protein ratio <0.5 Glucose <60 mg/dL LDH <200

Correct Answer ( C ) Explanation: Given his extensive smoking history, irregular mass noted on chest X-ray, and his recent unintentional weight loss, night sweats, and chronic cough, his pleural effusion is most likely secondary to an underlying lung malignancy. A pleural effusion is defined as more than 15 cc of fluid between the parietal and visceral pleuras. The most common causes are congestive heart failure (CHF), pneumonia, and malignancy. An effusion can be classified as transudative or exudative. A transudative effusion occurs secondary to increased hydrostatic pressure or decreased oncotic pressure. Common causes of transudative effusions include CHF, hypoalbuminemia, cirrhosis, and nephrotic syndrome. An exudative effusion is caused by increased permeability of pleural surfaces or decreased lymphatic flow. Common causes of exudative effusions include infection, such as pneumonia or tuberculosis, neoplasm, connective tissue disease, pulmonary embolism, uremia, pancreatitis, or esophageal rupture. A thoracentesis can be performed to analyze the fluid and determine a cause. Light's Criteria defines a transudative effusion by a LDH <200 (or less than 2/3 of the upper limit of the normal serum LDH level), a fluid:blood LDH ratio <0.6, and a fluid:blood protein ratio <0.5. In an exudative effusion, the glucose level is <60 mg/dL. In addition, if an exudative effusion is suspected, other tests such as fluid amylase, cell count, gram stain, culture, and cytology should be obtained. Fluid:blood LDH ratio <0.6 (A), fluid:blood protein ratio <0.5 (B), and LDH<200 (D) are all representative of a transudative effusion.

An 8-year-old boy was playing in his father's garden-shed and stepped on a nail. At that time, his father cleansed and dressed the wound appropriately. However, seven days later, the boy presents with difficulty walking. Examination shows a lateral sole puncture wound that is associated with surrounding warmth, erythema and edema. He also has an antalgic gait. Distal neurovascular examination is normal. An AP radiograph shows a round cuboid lucency. Which of the following is the most appropriate intervention at this time? Bone scan Oral antibiotics Surgical debridement Topical antibiotic ointment

Correct Answer ( C ) Explanation: In children, acute osteomyelitis commonly occurs with hematogenous spread or direct contamination, as in stepping on a nail. Pain, edema, erythema, warmth and generalized malaise are common symptoms of acute osteomyelitis. It is also important to consider cellulitis and septic arthritis in any case of suspected osteomyelitis. Laboratory investigation includes CBC, CRP and ESR, all of which may be normal early in the disease course. Blood cultures, which reveal the causative organism in up to 50% of cases, should also be obtained. Radiographs are also recommended. Early films may only show soft tissue swelling, but are useful in ruling-out fracture or neoplasm. Follow-up radiographs around a week after the injury usually shows periosteal elevation and bone lucency, two signs of bony destruction and significant pathology. In addition to blood cultures, aspiration is recommended if one can identify involvement of a specific bone and neoplasm has been ruled-out. Intravenous antibiotics are started immediately. Surgical debridement, especially in the case of nail puncture wounds, is strongly recommended, as failure to completely eradicate infection with Pseudomonas aeruginosa, the most common organism in nail puncture wounds, can lead to chronic refractory osteomyelitis, additional joint destruction, limb-length discrepancies, limb dysfunction and pathological fracture. A bone scan (A) or MRI may be considered for unclear cases of osteomyelitis. Timely, aggressive debridement trumps bone scanning in any case of highly suspected osteomyelitis, as is evident in the above patient presentation. Intravenous, not oral or topical (B and D), antibiotics are recommended in suspected acute osteomyelitis.

A 14-month-old boy presents to the emergency department with irritability. His parents report episodes of irritability that have been occurring every 2 hours over the past 12 hours. The episodes last for about 15 minutes then seem to resolve. He also seems sleepier than usual. His last stool was 2 days prior. On examination, the infant appears lethargic. He has mild abdominal distension and has involuntary guarding. Which of the following is the best diagnostic test? CT scan of the abdomen Hemoccult of the stool Ultrasound of the abdomen X-rays of the abdomen

Correct Answer ( C ) Explanation: Intussusception is best diagnosed by an abdominal ultrasound. Intussusception is a type of intestinal obstruction that occurs when part of the bowel telescopes into a nearby segment. It is most commonly seen in children aged 3 months to 6 years, and typically involves the ileocolic segment of bowel. Infants and children present with severe, intermittent abdominal pain and may develop into lethargy. Prolonged intussusception may result in shock. Affected children may have a recent history of emesis, and may have stools with blood and mucus. Once the obstruction occurs, stooling decreases. Diagnosis should be made by abdominal ultrasound, which has a sensitivity of 98-100% and a specificity of 88%. Ultrasound will show a tubular mass in longitudinal views, with a target sign in transverse views. Treatment consists of air, saline, or occasionally water-soluble contrast enemas. Complications of intussusception include bowel infarction, peritonitis, and death. CT scans (A) can show abdominal masses, organomegaly, infection, free fluid in the abdomen, and many other pathologies. The risk from radiation exposure does not warrant a CT scan when an abdominal ultrasound is likely to be diagnostic. If performed, a CT scan may identify a lead point for intussusception (present in 2-8% of patients), such as polyps, cysts, or tumors. Hemoccult of the stool (B) will be positive in the majority of cases. However, this is not a specific test, and the diagnosis should be confirmed with ultrasound. Radiographs of the abdomen (D) may show a density in the area of the intussusception. However, this is not as specific or sensitive as an ultrasound, and carries the risk of radiation exposure.

A 7-year-old girl presents to urgent care complaining of vaginal irritation over the last several days. Mom noticed that the patient's vaginal area is abnormally red and irritated and that her underwear has been slightly damp. The patient denies bleeding, urinary complaints, or abdominal pain. On exam, the patient's vulva appears erythematous and a small amount of whitish discharge is present. There is no foul odor, bleeding, or abnormal lesions present. Which of the following is the most likely cause of the patient's presentation? Bacterial vulvovaginitis Foreign body Irritant vulvovaginitis Sexual abuse

Correct Answer ( C ) Explanation: Irritant or nonspecific vulvovaginitis is the most common cause of vulvar irritation in prepubertal girls. Patients may present with or without a thin, whitish, non-foul-smelling discharge. Vulvar irritation may occur as the result of poor hygiene, such as wiping from back to front or not wiping adequately after urination, or secondary to chemical irritants, such as bubble bath, perfumed soaps, or clothes washed in strong detergent. Tightly fitting clothing such as leotards, bathing suits, tights, spandex, and nylon underwear can also be irritants. Factors which may increase the overall risk of vulvovaginitis include obesity, a relatively alkaline pH, thin vaginal mucosa, and small labia minora. Vulvovaginitis can be both treated and prevented by adopting thorough hygiene measures, avoiding bubble baths, and keeping soap away from direct contact with the genital area. Wearing 100% cotton underwear, avoiding tight clothes, and quickly removing wet bathing suits should be advised. Bacterial vulvovaginitis (A) typically presents acutely with foul-smelling vaginal discharge associated with erythema, burning, itching, and soreness. A vaginal foreign body (B) is suspected in patients who present with purulent discharge, vaginal bleeding, or foul odor, or in cases of chronic recurrent vulvovaginitis. The most common vaginal foreign body is toilet paper. A moistened swab or gentle irrigation may be sufficient to expel the material. However, some patients require examination under sedation or anesthesia to remove the object. Sexually transmitted diseases may be acquired perinatally. However, in the prepubertal girls they are most commonly the result of sexual abuse (D). Pathogens may include Chlamydia trachomatis, Neisseria gonorrheae, human papillomavirus, Treponema pallidum, and herpes simplex virus. A child suspected to be a victim of sexual abuse should have a thorough evaluation in an appropriate environment such as a child abuse assessment center.

What is the most common cause of an intracranial neoplasm? Astrocytoma Meningioma Metastases Pituitary adenoma

Correct Answer ( C ) Explanation: Metastatic disease (primarily from lung cancer) is the most common cause of an intracranial neoplasm. Other cancers that commonly metastasize to the brain include breast and colon carcinoma. The malignant gliomas (A) (anaplastic astrocytomas and glioblastoma multiforme) are the most common glial tumors. These are located in the cerebral hemispheres. Meningiomas (B) are generally benign, slow-growing tumors that originate in the meninges. Pituitary adenomas (D) are tumors originating from the pituitary gland and are often first noted when the patient exhibits visual impairment from compression of the optic chiasm.

A 34-year-old man presents with a purulent skin infection. Which of the following oral antibiotics will cover community-acquired methicillin-resistant Staphylococcus aureus (MRSA)? Amoxicillin Cephalexin Doxycycline Vancomycin

Correct Answer ( C ) Explanation: Methicillin-resistant Staphylococcus aureus (MRSA) is a strain of staphylococcus that has developed resistance to beta-lactam antibiotics. This resistant strain is often healthcare-associated, but is also found in the community. For limited community-acquired infections not requiring hospitalization, patients may be treated with oral antibiotics. A tetracycline (such as doxycycline) is an acceptable choice to treat community-acquired MRSA. Clindamycin or a sulfa drug (such as TMP-SMX) is another acceptable alternative. Healthcare-associated strains of MRSA are typically not susceptible to these oral antibiotics and will require IV vancomycin. Vancomycin is a bacteriostatic drug that inhibits cell-wall biosynthesis. Amoxicillin (A), an aminopenicillin beta-lactam antibiotic, and cephalexin (B), a cephalosporin beta-lactam antibiotic, are both resistant against MRSA. Vancomycin (D) has good activity against MRSA and is the IV antibiotic of choice for MRSA infection, but the PO form has very little GI absorption and is not helpful for MRSA treatment as an outpatient.

A 19-year-old man is working in Chicago for the Mercantile Exchange and develops mildly painful swelling over his parotid glands. On presentation to his primary care physician, a week later, he has a low-grade fever, earache, mild headache, and mild testicular pain. He denies sexual activity, and a few of his coworkers have similar symptoms. On physical exam, his physician finds the bilateral parotid pain with boggy edematous parotid tissue, minimal adenopathy, mild presternal edema, moderate abdominal pain over the pancreas, and mild testicular pain. Which one of the following is the most likely diagnosis in this young man? Atypical mycobacterial infection Epstein-Barr virus infection Mumps virus infection Parainfluenza virus infection

Correct Answer ( C ) Explanation: Mumps virus infection, in an unimmunized individual or in an individual who has not had a protective response to the vaccine, presents generally with acute parotid gland inflammation and swelling, and the pain often radiates up to the ear. It is a systemic disease with low-grade fever, involving other glandular tissue, including the pancreas and testicles. It can also be associated with mild CNS involvement. Atypical mycobacterial infection (A) often involves the parotid tissue and associated lymphoidal tissue around the parotid gland. It does not present acutely but rather sub-acutely or chronically in association with small hard lymph nodes and sometimes infection of the overlying skin. Epstein-Barr virus infection (B) may rarely present subacutely, with swelling of the parotid gland and associated lymph nodes but most frequently is associated with severe pharyngitis, generalized lymphadenopathy, hepatosplenomegaly, and atypical lymphocytosis. Parainfluenza virus (D) type 3 infection is a rare cause of parotitis. However, it is generally manifest with laryngotracheobronchitis (croup), pneumonia, or rhinopharyngitis.

A 35-year-old woman with alcohol abuse presents to clinic complaining of "skipped heart beats." Her examination is normal. An in-office ECG is also normal. Laboratory testing is nonspecific. Which of the following is the next most appropriate test? Echocardiogram Electromyography Holter monitor Sestamibi scan

Correct Answer ( C ) Explanation: Premature atrial contractions (PACs) are extra beats that originate outside the sinus node from ectopic atrial pacemakers. They appear interspersed throughout an underlying rhythm, usually sinus. Common symptoms include palpitations, described by many as a skipped beat or a missing beat. Others describe it as a stopping or flipping of the heartbeat. These symptoms mimic those of premature ventricular contractions. Causes include psychological stress, hypertension, valvulopathy, ischemic cardiac disease, stimulants (caffeine, tobacco, alcohol), digitalis toxicity, electrolyte abnormalities, and idiopathic. Most cases are asymptomatic and occur in otherwise healthy hearts. Many times one can only find them with continuous 24 hour monitoring via a Holter monitor. PACs are benign, however, a small amount may deteriorate into atrial flutter or fibrillation as well as supraventricular tachycardia. Although echocardiography (A) is usually part of the evaluation of PACs, one must first diagnose them by finding them on electrocardiography. Electromyography (B) is used in the evaluation of nerve and skeletal muscle disorders, not cardiac disorders. Sestamibi scanning (D) is a nuclear medicine test used in the evaluation of myocardial ischemia and infarction. It is not the next most appropriate test when the clinician does not have a working diagnosis of "skipped beats."

A 23-year-old woman presents to the office with plaques on the extensor surfaces of her knees and elbows. She complains that the areas are itchy and they bleed after they are scratched. On exam, you also notice pitting of the fingernails. What is the most likely diagnosis? Erythema multiforme Lichen simplex chronicus Psoriasis Seborrheic Dermatitis

Correct Answer ( C ) Explanation: Psoriasis is an autoimmune condition with peak incidence in the early 20's. It occurs equally in males and females, and has a hereditary component. Stereotypical lesions are chronic, relapsing, thick silvery and scaly papules and plaques that overly an erythematous base. It is a chronic disorder but certain triggers such as infection, drugs, and trauma can cause flares. The Koebner phenomenon occurs when physical trauma elicits new growth of psoriatic patches. The most common areas affected are the extensor surfaces of the extremities (elbows and knees), lower back, palms and soles, scalp, gluteal cleft, and genital regions. Management is with glucorticoid topicals, UV-A therapy, and, in extreme cases, methotrexate and biologic medications. Erythema multiforme (EM) (A) is a cutaneous reaction to various triggers, most commonly herpes simplex virus. EM is also triggered by medications such as sulfonamides, phenytoin, and penicillin. Lesions are erythematous flat papules with a central vesicle, often described as target lesion. Lichen simplex chronicus (B) is a skin condition, which results from chronic rubbing or scratching. Skin appears thickened, skin markings are accentuated, and there is minimal scaling. This condition can occur anywhere on the body. Seborrheic dermatitis (D) can occur at any age, but is commonly seen in infancy and old age. Skin lesions are characterized by redness and scaling with a greasy appearance. It occurs on the scalp (in baby's referred to as cradle cap), but also occurs in the eyebrows, lashes, nasolabial folds, and body folds.

In which biochemical abnormality can you see Chvostek sign (contraction of the facial muscles after tapping on the facial nerve anterior to the ear)? Hypercalcemia Hyperkalemia Hypocalcemia Hypokalemia

Correct Answer ( C ) Explanation: Signs of hypocalcemia include the Chvostek sign (evoked facial nerve irritability), Trousseau sign, and a prolonged QT interval. Severe hypocalcemia is associated with impaired cardiac contractility. In some individuals, hypoparathyroidism is detected only by an asymptomatic reduction in the serum calcium concentration. Hypocalcemia and an elevated serum phosphorus concentration in association with absent PTH levels confirm the diagnosis of hypoparathyroidism. In pseudohypoparathyroidism, parathyroid hormone (PTH) levels are elevated, reflecting the PTH-resistant state, but otherwise the biochemical findings of hypocalcemia and hyperphosphatemia are similar to those of hypoparathyroidism. The mnemonic "stones," "bones," "abdominal moans," and "psychic groans" describes the constellation of symptoms and signs of hypercalcemia (A). Patients with hyperkalemia (B) may be asymptomatic, or they may report the following symptoms although cardiac and neurologic symptoms predominate: fatigue, weakness, paralysis, palpitations. Hypokalemia (D) is not associated with Chvostek's sign. It can lead to weakness, fatigue, constipation, u waves on the ECG, and cardiac dysrhythmia.

A 16-year-old girl is seen in the emergency room after a new, generalized tonic-clonic seizure. She has no history of seizures and recently moved from the Caribbean with her family. An MRI of the head shows multiple calcified cysts in the parieto-occipital region. Which of the following is the most likely infection in this child? Ascariasis Hookworm disease Neurocysticercosis Strongyloidiasis

Correct Answer ( C ) Explanation: Taenia solium, the pork tapeworm, is transmitted to humans by ingestion of food or water contaminated with the eggs. In the small intestine, the egg releases an oncosphere that crosses the gut wall and spreads hematogenously to the brain leading to neurocysticercosis. Consumption of undercooked pork produces an intestinal infection that is caused by the adult worm. Individuals infected with an adult worm can infect themselves or others with the eggs via fecal-oral route. Neurocysticercosis should be suspected in any child with new onset seizures and a history of living in an endemic area or exposed to someone from an endemic area. MRI of the head is the most useful test given that it provides information about cyst viability and associated inflammation. Albendazole is the pharmacologic drug choice however; antiparasitic agents are generally not used as most cysts resolve spontaneously over time. Ascariasis (A) is an intestinal infection that does not lead to neurological problems. Hookworm disease (B) is known to cause severe anemia as well as intra-abdominal symptoms. Strongyloidiasis (D) is caused by the organisms in contaminated water that are capable of penetrating human skin. It does not cause neurological disease.

A patient is being evaluated for palpitations in the emergency department. An ECG reveals a short PR interval, widened QRS and slurred upstroke of the QRS complex. Which of the following is the name of bypass pathway associated with this condition? Bachmann's bundle Bundle of His Bundle of Kent Left bundle branch

Correct Answer ( C ) Explanation: The bundle of Kent is the bypass pathway associated with Wolff-Parkinson-White syndrome, which is the most likely diagnosis in this patient. Wolff-Parkinson-White syndrome is a pattern of pre-excitation on ECG accompanied by a symptomatic tachycardia. It results from an accessory pathway that directly connects the atria and ventricles and bypasses the AV node. The bypass pathways are accessory connections called Kent bundles between the atrium and ventricle. If a bypass pathway conducts antegrade, a pre-excitation pattern may be seen on the ECG. This includes a short PR interval because the bypass pathway conducts rapidly and faster than the AV node; a delta wave, which is a slurred initial segment of the QRS complex resulting from slow conduction through ventricular tissue instead of the His-Purkinje system; and a widened QRS that consists of fusion between early ventricular activation from pre-excitation with the later ventricular activation resulting from transmission through the AV node. Bachmann's bundle (A) is a branch in the atrial conduction system, which facilitates right to left atrial conduction. The bundle of His (B) is specialized conduction tissue that transmits the electrical impulses from the AV node to the point of the apex of the fascicular branches via the left and right bundle branches. The left bundle branch (D) and right bundle branch then lead to the Purkinje fibers, which provide electrical conduction to the ventricles, causing the cardiac muscle of the ventricles to contract at a paced interval.

A postmenopausal patient of yours is diagnosed with atrophic vaginitis and dyspareunia. She denies other menopausal complaints. Vaginal lubricants and moisturizers are not helping to decrease her symptoms. Which of the following would be most appropriate to prescribe next? Oral estrogen Oral progesterone Vaginal estrogen Vaginal progesterone

Correct Answer ( C ) Explanation: The symptoms of vaginal atrophy, or atrophic vaginitis, can be quite bothersome and uncomfortable. They include irritation, burning, bleeding, dryness and dyspareunia, and are common in postmenopausal women. Treatment goals are based on relief of these symptoms. Regular sexual activity can help keep vaginal epithelium healthy. First line therapy includes vaginal lubricants and moisturizers. If patients fail these, consider vaginal estrogen instead of oral estrogen preparations. Oral estrogen (A) should be reserved for systemic menopausal symptoms other than vaginal dryness, such as hot flashes. Progesterone (B & D) is not used to improve the lubrication or elasticity of the vaginal epithelium.

A 65-year-old man with hypertension and diabetes presents with chest pain for 2 days. He states the pain is sharp, burning and severe. You note the findings on the image above. What management is indicated? Admission for serial troponins Intravenous acyclovir for 7 days Oral acyclovir for 7 days Prednisone for 5 days

Correct Answer ( C ) Explanation: This patient presents with herpes zoster more commonly referred to as shingles. Zoster results from reactivation of latent varicella-zoster virus (VZV) in cranial nerve or dorsal-root ganglia with spread along the sensory nerve to the dermatome. The major risk factor for herpes zoster is increasing age as there is a decline in T-cell immunity. Although herpes zoster can progress to a systemic infection, particularly in those with immunocompromised states, the major complication is postherpetic neuralgia. Postherpetic neuralgia can be severe and debilitating. Antiviral therapy (usually with acyclovir or valacyclovir) is recommended in all immunocompromised patients and selected groups of nonimmunocompromised patients as seen in the image below. Antiviral agents hasten the resolution of lesions, reduce the formation of new lesions, reduce viral shedding and decrease the severity of acute pain. Therapy should be started as soon as possible and efficacy decreases after 72 hours of symptoms. This patient's chest pain does not appear to be cardiac in origin and serial troponins (A) are not indicated. The use of glucocortocoids (D) for the treatment of herpes zoster remains controversial. Some studies have shown a reduction in acute pain and accelerated early healing. Because they are immunosuppressive, steroids should never be given alone but only in conjunction with antiviral therapy. Additionally, they should be avoided in patients with hypertension, diabetes mellitus, peptic ulcer disease and in the elderly all of whom are at increased risk of adverse events. Intravenous antivirals (B) should be considered in patients with refractory disease, multiple dermatomes (especially if they cross the midline), ocular involvement, systemic symptoms and in immunocompromised individuals.

A 63-year-old man with hypertension and dyslipidemia presents with pain and swelling of his left great toe. He denies trauma or fever. Examination reveals an exquisitely tender and swollen first metatarsophalangeal (MTP) joint on his left foot. Arthrocentesis yields fluid as shown above. What is the first line therapy for this patient? Acetaminophen Allopurinol Ibuprofen Probenecid

Correct Answer ( C ) Explanation: This patient presents with monoarticular arthritis and a synovial fluid sample consistent with gout. Gout is a systemic disorder that manifests with joint inflammation. It is caused by precipitation of uric acid crystals from extracellular fluid. Hyperuricemia results either from underexcretion of end products of purine metabolism from the kidney or overproduction (more rare). During an episode of gout, polymorphonuclear cells ingest the crystals and release cytokines leading to an inflammatory reaction within the synovium. The most commonly affected joint is the first metatarsophalangeal (MTP) joint followed by the knee. It is important to consider that patients with gout are at a higher risk of developing septic arthritis because the joints are chronically damaged. In patients with a history of gout that present with symptoms concerning for septic arthritis (pain, fever, decreased range of motion) arthrocentesis should be performed to rule out the presence of infection. Management of gout should be split into acute gouty attack treatment and long-term prophylaxis. The mainstay of acute therapy is NSAIDs and colchicine. Indomethacin, naproxen and ibuprofen all may be used in acute treatment. Colchicine inhibits microtubule formation reducing the inflammatory response to uric acid crystals. Unfortunately, the drug has a number of side effects that are almost universally experienced at therapeutic doses (nausea, vomiting, diarrhea). Acetaminophen (A) does not have anti-inflammatory properties and offers little benefit in acute gouty attacks. Allopurinol (B) is a xanthine oxidase inhibitor that prevents production of uric acid. It is useful in patients with increased synthesis and decreased clearance of uric acid. Allopurinol should not be started at the time of an acute attack of gout because it can lead to a transient increase in uric acid levels. However, patients already prescribed allopurinol should continue to take it at the same dose during acute episodes. Probenecid (D) is useful in prophylaxis but not for acute episodes.

An elderly man presents with 6 months of progressive positional dyspnea. Nine months ago, he had a mild heart attack and was properly treated. Today, you appreciate a new loud, blowing holosystolic murmur heard at the apex. Which of the following color Doppler echocardiography findings would you most expect to find based on these signs and symptoms? During diastole, blood flows from the right ventricle into the right atrium During diastole, blood is seen flowing from the aorta into the coronary arteries During systole, blood flows from the left ventricle into the aorta During systole, blood is seen flowing from the left ventricle into the left atrium

Correct Answer ( D ) Explanation: One of the most common cardiac valve abnormalities is mitral regurgitation (MR). Also known as mitral insufficiency, it can be caused by rheumatic fever, myocardial infarction, ruptured chordae or papillary muscles and cardiac myxomatous degeneration. In fact, the most common cause of mitral regurgitation is the myxomatous breakdown of this valve's connective tissue, a condition called mitral valve prolapse. Mitral prolapse can be heard at the apex and the left sternal border best when the patient is seated, and is typically described as a late systolic murmur preceded by an earlier systolic click. However, mitral regurgitation, also heard best at the apex is described as a holosystolic, plateau-shaped, loud blowing murmur, which may be associated with an apical thrill. An echocardiogram is used to confirm the diagnosis, the key finding of which is a stream of blood flowing out of the left ventricle into the left atrium during systole. Symptoms depend on presentation, acute or chronic. Acute MR includes dyspnea, orthopnea, pulmonary edema, poor exercise tolerance, palpitations and cardiogenic shock. Acute cases are mainly infectious in nature. Chronic compensated cases may be asymptomatic, whereas volume-overloaded decompensated cases will shows signs of congestive heart failure. Chronic cases are usually treated with ACE-inhibitors and hydralazine, the combination of which can effectively delay valve replacement surgery. Indications for surgery in chronic cases depends on the ejection fraction, the presence of new onset atrial fibrillation or severe pulmonary hypertension. Normally during diastole, the blood that was just pumped out of the left ventricle pushes back, closes the aortic valve and enters the ostia of the coronary arteries (B). Most of coronary blood flow occurs during diastole. During diastole, blood normally flows from the right atrium into the right ventricle (A), not vice-versa. During systole, blood normally flows out of the left ventricle into the aorta (C).

A 58-year-old man presents to the ED with complaints of severe fever and chills, a diffuse headache, abdominal cramping, nausea and vomiting, and general weakness with lethargy. He returned from a business trip to Central America six days ago and has been experiencing such symptoms every three days. His blood pressure is 148/72 mm Hg, respiratory rate 24, heart rate 122, oxygen saturation of 96% on 2L NC, and temperature 40.6°C. On exam, you note scleral icterus. Physical exam is unremarkable, with guaiac negative stool. Lab results reveal a hemoglobin of 8 g/dL, platelets 175/µL, LDH 800 U/L, and indirect bilirubin 2.5 mg/dL. Which of the following is the most likely cause of this patient's symptoms? Babesiosis Chagas disease Dengue fever Malaria

Correct Answer ( D ) Explanation: There are four known types of malaria: Plasmodium falciparum, Plasmodium vivax, Plasmodium ovale, and Plasmodium malariae. Although P. vivax is the most common form, P. falciparum leads to the most virulent disease, causing most cases of severe malaria and most malaria-related deaths. Because of its ubiquitous presence in much of the world, malaria should be considered in any patient with severe fevers and chills who reports recent travel to an endemic region. Classically, malaria begins with a flu-like prodrome that progresses to cyclical episodes of chills followed by fever, each lasting about two hours. These episodes recur every three days with P. vivax and P. ovale and every four days for P. falciparum and P. malariae. These episodes are associated with a hemolytic anemia, as identified in this patient by scleral icterus, low hemoglobin, elevated LDH, and indirect bilirubin. Identifying Plasmodial parasites on Giemsa-stained thick and thin smears makes the definitive diagnosis. Babesiosis (A) is caused by a malaria-like protozoan parasite that infects red blood cells and leads to hemolytic anemia. It is transmitted by the Ixodes tick (the same as Lyme disease). The tick's reservoir is deer and mice. Transmission to humans occurs throughout the United States, particularly in the Northeast, during the summer months. Chagas disease (B) is caused by Trypanosoma cruzi and transmitted by the bite of the reduviid (kissing) bug. It is associated with unilateral periorbital swelling at the site of the offending insect bite (chagoma), cardiomyopathy, and megacolon. The reduviid bug is found mainly in South and Central America. Dengue fever (C) is a viral infection seen in the tropics and subtropics transmitted by the bite of the Aedes mosquito. It causes an acute, self-limited illness defined by high fevers and severe myalgias ("breakbone fever"), along with headache, rash, lymphadenopathy, and leukopenia. A distinction between malaria and dengue hemorrhagic fever is that dengue hemorrhagic fever can cause an elevated hematocrit due to vascular permeability, whereas malaria causes a hemolytic anemia.

A 76-year-old man with a history of hypertension presents after a syncopal event. He also reports decreased exercise tolerance over the last two months. He is currently asymptomatic. His ECG is normal sinus rhythm with no changes. On examination, he has a harsh systolic murmur that radiates to the carotid arteries. Which of the following is true about the most likely cause of his syncope? Critical disease is defined by heart valve area less than 2 cm2 It typically results from rupture of the chordae tendinae The murmur increases with valsalva Vasodilators and diuretics should be used with caution

Correct Answer ( D ) Explanation: Aortic stenosis is an abnormality of the heart valve that prevents left ventricular outflow. This results in left ventricular hypertrophy which eventually impairs diastolic filling and results in increased myocardial oxygen demand. The most common cause is degenerative calcification associated with diabetes, hypertension, and advanced age. Exam findings include a harsh systolic murmur that radiates to the carotids as well as an S4 gallop. Carotid pulses may be delayed and diminished and a narrowed pulse pressure is typically present. Classic symptoms start with dyspnea and chest pain and progress to syncope and congestive heart failure. Although patients may be asymptomatic for a long period of time, once symptoms develop mortality increases significantly. Because patients with critical aortic stenosis are preload dependent, vasodilators and diuretics should be avoided as they can cause significant hypotension. Definitive treatment is aortic valve replacement. A normal aortic valve area is greater than 3 cm2. Critical aortic stenosis (A) is defined by a heart valve area of less than 0.8 cm2. Mitral regurgitation is most commonly caused by post-myocardial infarction rupture of the chordae tendinae (B) or papillary muscle dysfunction and presents with acute onset of pulmonary edema and midsystolic apical murmur. The murmur of aortic stenosis decreases with valsalva (C).

Which of the following cerebrospinal fluid analyses is most consistent with viral meningitis? WBC 2900, glucose 20, protein 95 WBC 350, glucose 15, protein 190 WBC 400, glucose 20, protein 270 WBC 525, glucose 75, protein 90

Correct Answer ( D ) Explanation: Normal adult cerebrospinal fluid (CSF) contains no more than 5 leukocytes/µL. More than 5 cells/µL should be considered evidence of CNS infection. Below is a table representing the typical spinal fluid results for meningeal processes. Although the diagnosis of viral meningitis is often straightforward, there can be an overlap of CSF findings in patients with early and partially treated bacterial meningitis, making the distinction somewhat difficult. Moreover, while neutrophils tend to predominate in bacterial meningitis, they may be present in the CSF for the first 24 hours with viral meningitis. An elevated WBC with low glucose and elevated protein (A) is characteristic of bacterial meningitis. There will also be a predominance of neutrophils, and the Gram stain will generally be positive. Fungal meningitis and tubercular meningitis have similar CSF findings. A WBC between 10 - 1,000 with high protein and low glucose (B) is most consistent with fungal meningitis. This is associated with lymphocyte predominance and a positive fungal culture. A WBC between 10 - 1,000 with very high protein and low glucose (C) is most consistent with tubercular meningitis. This is associated with lymphocyte predominance and a positive AFB culture.

45-year-old woman presents with nonradicular, flexion-based back pain that began atraumatically three weeks ago. Her medical history is only significant for hypertension and nephrolithiasis. She denies numbness, weakness, fever, bowel or bladder changes nor a history of cancer. Physical exam is unremarkable. Which of the following is the next best step? Order a lumbar MRI with contrast Order a neurosurgical consultation for lumbar spine evaluation Recommend bed rest for five days Recommend continued activity within the limits of the pain

Correct Answer ( D ) Explanation: This patient has acute, nonspecific back pain (lumbago). Lower back pain affects 60% to 80% of adults at some point in their lives. Most cases resolve within a few weeks so management is generally conservative. Patients should be instructed to return to work and continue with daily activity based on their pain tolerance. Engaging in activity within the limits of pain aids recovery. Red flags that raise suspicion for a less benign process include night pain and weight loss (suspect neoplasm); fevers, chills, and sweats (spinal infection); acute bony tenderness (fracture); morning stiffness lasting >30 minutes in young adults (seronegative spondyloarthropathy); and any neurologic deficit or bowel or bladder involvement (nerve root compression). Imaging (A) for nonspecific low back pain is low yield and costly and is not recommended. Surgical consultation (B) should be considered if a patient fails conservative treatment, or develops progressive and functionally limiting neurological symptoms such as leg weakness or bowel or bladder incontinence. Prolonged bed rest greater than two days (C) does not hasten healing and may lead to a worse prognosis.

A 3-year-old boy with no past medical history presents with a seizure. His mom states that he had a cough and a fever for the last day and then she witnessed him having 45 seconds of tonic clonic movements. In the Emergency Department, the patient has normal vital signs except for a temperature of 102°F. Physical examination reveals a well-appearing boy who is playful and interactive. He has no neurologic findings. What management is indicated? Admit for electroencephalography Lumbar puncture Noncontrast CT scan of the head Observation, reassurance, and follow up with pediatrician

Correct Answer ( D ) Explanation: This patient presents with a simple febrile seizure and the parents should be reassured and instructed to follow up with the child's pediatrician. Febrile seizures are defined as a seizure occurring in the presence of a fever without central nervous system (CNS) infection or other cause. A simple febrile seizure describes a brief (<15 minutes) generalized seizure in a developmentally normal child between the ages of 6 months and 5 years. A complex febrile seizure occurs when there are multiple seizures during the same illness, the seizures are prolonged or focal. Febrile seizures are common and occur in 2-5% of children. Children who experience a febrile seizure are likely to have a recurrence (30%) and at a 2-3% risk of developing epilepsy. Seizure activity usually ceases on its own without specific management but if the child has a seizure in the ED, they can be treated with benzodiazepines. Children with simple febrile seizures do not require any specific testing beyond a good history and physical examination. Administration of antipyretics has not been found to reduce the likelihood of recurrence. Reassurance and follow up with their pediatrician are the most important aspects of management. Admission for EEG monitoring (A) is unnecessary after a simple febrile seizure. Likewise, a CT of the head (C) and lumbar puncture (B) are not necessary if the patient's physical examination is unremarkable.


Ensembles d'études connexes

Final Review - CIST1601-Information Security Fund

View Set

Module 1.01: Natural Rights - Quiz

View Set

Chapter 26: Disorders of Blood Flow and Blood Pressure Regulation Patho Prep U

View Set

Pathology-Chapter 3, Musculoskeletal System Conditions

View Set